Inequality

(ab+bc+ca)(1(a+b)2+1(b+c)2+1(c+a)2) \large (ab+bc+ca) \left (\frac 1{(a+b)^2} + \frac 1{(b+c)^2}+\frac 1{(c+a)^2} \right)

For a,b,ca, b, c are positive reals, if the infinium (minimum) value of the expression above can be expressed as xy \frac{ x}{y} for positive coprime integers xx and yy, find the value of xy x - y .

This problem is already posted but it has no solutions and I wanted to know it and so I am posting it again.

#Algebra

Note by Ankit Kumar Jain
4 years, 4 months ago

No vote yet
1 vote

  Easy Math Editor

This discussion board is a place to discuss our Daily Challenges and the math and science related to those challenges. Explanations are more than just a solution — they should explain the steps and thinking strategies that you used to obtain the solution. Comments should further the discussion of math and science.

When posting on Brilliant:

  • Use the emojis to react to an explanation, whether you're congratulating a job well done , or just really confused .
  • Ask specific questions about the challenge or the steps in somebody's explanation. Well-posed questions can add a lot to the discussion, but posting "I don't understand!" doesn't help anyone.
  • Try to contribute something new to the discussion, whether it is an extension, generalization or other idea related to the challenge.
  • Stay on topic — we're all here to learn more about math and science, not to hear about your favorite get-rich-quick scheme or current world events.

MarkdownAppears as
*italics* or _italics_ italics
**bold** or __bold__ bold

- bulleted
- list

  • bulleted
  • list

1. numbered
2. list

  1. numbered
  2. list
Note: you must add a full line of space before and after lists for them to show up correctly
paragraph 1

paragraph 2

paragraph 1

paragraph 2

[example link](https://brilliant.org)example link
> This is a quote
This is a quote
    # I indented these lines
    # 4 spaces, and now they show
    # up as a code block.

    print "hello world"
# I indented these lines
# 4 spaces, and now they show
# up as a code block.

print "hello world"
MathAppears as
Remember to wrap math in \( ... \) or \[ ... \] to ensure proper formatting.
2 \times 3 2×3 2 \times 3
2^{34} 234 2^{34}
a_{i-1} ai1 a_{i-1}
\frac{2}{3} 23 \frac{2}{3}
\sqrt{2} 2 \sqrt{2}
\sum_{i=1}^3 i=13 \sum_{i=1}^3
\sin \theta sinθ \sin \theta
\boxed{123} 123 \boxed{123}

Comments

Why don't you link us to the problem itself?

What have you tried? Have you tried homogenizing a,b,ca,b,c? That is, set one of the symmetric sums of a,b,ca,b,c to be a certain constant.

Pi Han Goh - 4 years, 4 months ago

Sir, I don't know how to create a link. Can you please explain me what are you trying to convey through 'homogenizing a,b,ca, b, c'?

Ankit Kumar Jain - 4 years, 4 months ago

Log in to reply

I've fixed the link for you.

Synopsis of homogenizing the variables: If the minimum value occurs when (a,b,c)=(a,b,c)(a,b,c) = (a', b' , c') , then the minimum value also occurs when (a,b,c)=(ka,kb,kc)(a,b,c) = (ka', kb', kc') , where kk is any positive number.

So in this case, you can safely make the assumption that a+b=c=La + b = c = L , ab+ac+bc=Lab + ac + bc = L or abc=Labc = L for some positive constant LL .

Now suppose I assumed that a+b+c=La+b+c = L , can you simplify the desired expression in terms of LL ? What classical inequalities can we use? Maybe Cauchy-Schwarz inequality? Maybe Titu's lemma?

Pi Han Goh - 4 years, 4 months ago

Log in to reply

Sir , I tried doing it but couldn't crack it.

Ankit Kumar Jain - 4 years, 4 months ago

Log in to reply

@Ankit Kumar Jain What inequalities have you tried? What haven't you tried? Are you familiar with all (or most) the common classical inequalities?

Pi Han Goh - 4 years, 4 months ago

Log in to reply

@Pi Han Goh Sir , I couldn't simplify the expressions in terms of L to make it something elegant.

As for the inequalities , I know almost all of them.

I tried solving it directly using Cauchy-Schwarz inequality.

Ankit Kumar Jain - 4 years, 4 months ago

Log in to reply

@Ankit Kumar Jain Sketch proof: use CS on (1/(a+b)^2 , 1/(a+c)^2 , 1/(b+c)^2 ) with (1,1,1)

Then apply AM-HM for the numbers (1/(a+b) + 1/(b + c) + 1/(a+c))

WLOG, set a+b+c = 1

What's left is to minimize (ab+ac+bc) subject to a,b,c>0 and a+b+c=1.

Use Lagrange multipliers to finish it off, and you will get min(ab+ac+bc) occurs when a=b=c

Pi Han Goh - 4 years, 4 months ago

Log in to reply

@Pi Han Goh I am familiar with the classical inequalities but not with Lagrange Multipliers...I even tried the wiki but couldn't really make it.

Ankit Kumar Jain - 4 years, 4 months ago

Log in to reply

@Ankit Kumar Jain I'll post my complete solution in the weekends...

Pi Han Goh - 4 years, 4 months ago

@Ankit Kumar Jain Wait. Are you able to simplify the problem to "Minimize(ab+ac+bc) subject to a,b,c>0 and a+b+c=1" ?

If yes, I'll just post the calculus approach here and you can submit the full solution in the problem itself. How about that?

Pi Han Goh - 4 years, 3 months ago

Log in to reply

@Pi Han Goh Yes , I could do that.

Ankit Kumar Jain - 4 years, 3 months ago

Log in to reply

@Ankit Kumar Jain [This is wrong, I've made a mistake somewhere, but I can't seem to fix it....]

We want to minimize ab+bc+acab + bc + ac subject to the constraints a,b,c>0a,b,c> 0 and a+b+c=1a+b+c= 1.

Substituting c=1abc = 1 -a-b into this given expression gives a+ba2b2aba + b - a^2 - b^2 - ab .

Let f=a+ba2b2abf = a+b - a^2 - b^2 - ab . Since both a,ba,b are in the open interval (0,1)(0,1) , then ff has no boundary points. So the min/max of ff must occur at its extremal point(s).

Taking the partial derivative of ff with respect to aa, we get fa=12abf_a = 1 - 2a - b .

Similarly, fb=12baf_b = 1 - 2b - a .

At the extremal point(s), fa=fb=0f_a = f_b = 0 . Solving this gives a=b=13f=13a = b= \dfrac13\Rightarrow f = \dfrac13.

Now, we want to prove that f=13f = \dfrac13 is a minimum value. To do so, we apply the second derivative test determinant:

D=faafbb(fab)2, D = f_{aa} f_{bb} - (f_{ab})^2 ,

where faa=2a2f=2f_{aa} = \dfrac{\partial^2}{\partial a^2} f = -2 , fbb=2b2f=2f_{bb} = \dfrac{\partial^2}{\partial b^2} f = -2 and fab=2abf=1f_{ab} = \dfrac{\partial^2}{\partial a \partial b} f = -1 .

This gives D=(2)(2)(1)2=3>0D = (-2)(-2) - (-1)^2 = 3 > 0 with faa(13,13)=13>0f_{aa} \left ( \dfrac13, \dfrac13\right) = \dfrac13 > 0 , thus the point we have found is indeed a minimum point.

In other words, we have shown that the expression ab+bc+acab+bc+ac is minimized when a=b=c=13a =b=c= \dfrac13 (under those stated constraints).

Pi Han Goh - 4 years, 3 months ago

Log in to reply

@Pi Han Goh Not relevant to this problem,

In the Weighted AM -GM Inequality , are the weights positive reals , fractions or positive integers ?

Ankit Kumar Jain - 4 years, 3 months ago

Log in to reply

@Ankit Kumar Jain positive reals.

Pi Han Goh - 4 years, 3 months ago

Log in to reply

@Pi Han Goh Sir, but logically going like weights are the number of times we are using the term , like if mm is weight of aa , then , it signifies a+a+a+a++ax times\underbrace{a + a + a + a + \cdot \cdot \cdot +a}_{\text{x times}}

Ankit Kumar Jain - 4 years, 3 months ago

Log in to reply

@Ankit Kumar Jain Oh sorry, I misread your question. I thought you're talking about power mean inequality.

My bad, the answer should be "positive integers".

Pi Han Goh - 4 years, 3 months ago

Log in to reply

@Pi Han Goh Sir , I was just reading the wiki on Young's Inequality , and its proof used Weighted AM - GM with weights as positive reals.

Ankit Kumar Jain - 4 years, 3 months ago

Log in to reply

@Ankit Kumar Jain Oh curses. I forgot. Yes, it's positive reals, not "positive integers'. Sorry I got it wrong twice!

Pi Han Goh - 4 years, 3 months ago

Log in to reply

@Pi Han Goh But as I said in my previous comment , how come the number of ways we are taking a term be a non - natural number?

Ankit Kumar Jain - 4 years, 3 months ago

Log in to reply

@Ankit Kumar Jain Sorry for the delay. I need to read up the articles for AMGM again.

Anyway, the reason why the weights can be non-integers as well is because AMGM is a derivation from the Jensen's inequality. And because Jensen's inequality doesn't specify that the weights need to be integers, then neither does AMGM as well.

Note that a^3.5 cannot be expressed as (a * a * a ... * a) (3.5 times), same goes for the AMGM stuffs that you've mentioned.

On the other hand, I've found a (correct) way of proving that min(ab + ac + bc) occurs when a=b=c.

Hint: Start with (a+b+c)^2 =a^2+b^2+c^2 + 2(ab+ac+bc), so we want to maximize (a^2 + b^2 + c^2) subject to the same constraints. You can either solve by Lagrange multipliers (followed by Hessian matrix) or interpret the expression (a^2+b^2+c^2) as an equation of a sphere and the equation (a+b+c=1) as an equation of a plane in the first quadrant.

I'm actually still not satisfied with my approach because it uses calculus approach, which is a little bit unacceptable in my opinion. I'll try to post a full inequalities approach (and no calculus method) if I'm able to.

Pi Han Goh - 4 years, 3 months ago

Log in to reply

@Pi Han Goh Sir , in the derivation of AM - GM Inequality , definitely , we'll have to take the function of the particular term like f(a)f(a) n number of times to get the power raised in the GM side and that number multiplied in the AM side, so won't that force nn to be a positive integer?

Ankit Kumar Jain - 4 years, 3 months ago

Log in to reply

@Ankit Kumar Jain Nope, not necessary. Like I said above, weighted AMGM is a derivation/simpler version of Jensen's, you shouldn't interpret the final weighted form of AMGM as though all the weights must be integers.

On the other hand, you can't guarantee that if you raised it "n number of times" then it will be an integer. You could have weights of irrational numbers like (sqrt2) as well, so no matter how what integer value of "n' you raise, you can't get an integer.

Pi Han Goh - 4 years, 3 months ago

Log in to reply

@Pi Han Goh I was reading the wiki on AM - GM Inequality just now and there I saw in the proof that to get the result :

a1+a2+a3++ann(a1a2a3an)1na_1 + a_2 + a_3 + \cdot \cdot \cdot + a_n \geq n\cdot {(a_1a_2a_3\cdot \cdot \cdot a_n)}^{\frac{1}{n}}

Jensen' s Inequality was used as :

Take f(x)f(x) as ln(x)ln(x) , then since f"(x)<0f"(x) < 0.

f(a1)+f(a2)+f(a3)++f(an)nf(a1+a2+a3++ann)\Rightarrow \frac{f(a_1) + f(a_2) + f(a_3) + \cdot \cdot \cdot + f(a_n)}{n} \leq f(\frac{a_1 +a_2 +a_3 +\cdot \cdot \cdot + a_n}{n}).

So to get the weighted result we'll have to do this (Suppose for two terms):

f(a1)+f(a1)+f(a1)++f(a1)b_1 times+f(a2)+f(a2)+f(a2)++f(a2)b_2 timesb1+b2f(a1+a1++a1b_1 times+a2+a2++a2b_2 timesb1+b2)\frac{\underbrace{f(a_1) + f(a_1) + f(a_1) + \cdot \cdot \cdot + f(a_1)}_{\text{b\_1 times}} + \underbrace{f(a_2) + f(a_2) + f(a_2) + \cdot \cdot \cdot + f(a_2)}_{\text{b\_2 times}}}{b_1 + b_2} \leq f(\frac{\underbrace{a_1 +a_1 + \cdot \cdot \cdot + a_1}_{\text{b\_1 times}} + \underbrace{a_2 + a_2 + \cdot \cdot \cdot + a_2}_{\text{b\_2 times}}}{b_1 +b_2})

So , won't this force bib_i to be a positive integer?

Ankit Kumar Jain - 4 years, 3 months ago

@Pi Han Goh Oh sorry!!I I understood my mistake. Thanks a lot.

Ankit Kumar Jain - 4 years, 3 months ago

@Pi Han Goh Sir, where am I at fault?

Ankit Kumar Jain - 4 years, 3 months ago

Here's the link

Ankit Kumar Jain - 4 years, 4 months ago

An easy one ans is 5

Md Zuhair - 4 years, 3 months ago

Log in to reply

Please post your complete solution in that case.

Ankit Kumar Jain - 4 years, 3 months ago

Log in to reply

Ya sure.. but currently my pc is not working. As soon as I get it back most probably by 3 days... I will post

Md Zuhair - 4 years, 3 months ago

Log in to reply

@Md Zuhair Okay! :)

Ankit Kumar Jain - 4 years, 3 months ago
×

Problem Loading...

Note Loading...

Set Loading...